27: The ratio is of the type . Since the highest powers prevail, the expression is for large values of n approximately

To prove it, factor out the dominant terms.

Check that if you use l'Hospital's rule (which is possible here), then it does not help at all, since you get pretty much the same expression.

Next hint
Answer